Category Theory: Inverse Limit in Sets

Click For Summary
The discussion focuses on demonstrating the existence of an inverse limit in the category of sets for a given index category and diagram. The problem involves showing that a unique morphism exists from a set W to the inverse limit A, satisfying the required properties of the morphisms w_i. Participants clarify that the maps w_i induce a unique map w into the product of sets, which must also map into A. The conversation emphasizes the importance of the universal property of products and the conditions under which the morphisms commute. The final consensus confirms the correctness of the proposed approach to the problem.
Mandelbroth
Messages
610
Reaction score
24
I think this looks like a homework problem, so I'll just put it here.

Homework Statement


Demonstrate that, for any index category ##\mathscr{J}## and any diagram ##\mathcal{F}:\mathscr{J}\to\mathbf{Sets}##,

$$\varprojlim_{\mathscr{J}}A_j=\left\{a\in \prod_{j\in \operatorname{obj}( \mathscr{J})}A_j~\vert~(i,k\in\operatorname{obj}(\mathscr{J}),~a_i\in A_i,~a_k\in A_k, \text{ and } \phi_{ik}\in \hom_{\mathscr{J}}(i,k))~\implies~a_k=\mathcal{F}(\phi_{ik})(a_i) \right\}=A$$
along with the obvious projections, which I'll denote ##l_i: A\to A_i##.

Homework Equations


I don't know how to make diagrams in TeX, so I'll just link to the universal property.

The Attempt at a Solution


Suppose ##W## has morphisms ##w_i: W\to A_i## that satisfy ##w_k=\mathcal{F}(\phi_{ik})\circ w_i##. We wish to show the existence of a unique morphism ##w: W\to A## such that ##w_i=l_i\circ w##.

My thought is that both ##W## and ##A## clearly map into the product ##\prod A_j##. We even have the unique map from ##A## to ##\prod A_j##, set inclusion, satisfying the universal property for the product. However, I don't know how to proceed. Any nudges in the right direction would be helpful. Thank you!
 
Physics news on Phys.org
The maps ##w_i:W\rightarrow A_i## induce a unique map ##w:W\rightarrow \prod_{i\in I} A## such that ##l_i\circ w = w_i##. This is essentially by definition of the product in the category.

Now show that ##w## actually maps into ##A##, that is, that ##w(W)\subseteq A## and that it satisfies the universal property.
 
  • Like
Likes 1 person
micromass said:
The maps ##w_i:W\rightarrow A_i## induce a unique map ##w:W\rightarrow \prod_{i\in I} A## such that ##l_i\circ w = w_i##. This is essentially by definition of the product in the category.

Now show that ##w## actually maps into ##A##, that is, that ##w(W)\subseteq A## and that it satisfies the universal property.
Let me see if I understand what you're saying. Let ##\pi_i: \prod A_j\to A_i## be the natural projection maps from the universal property defining the product. Then, ##\pi_i\circ w=w_i##. But, since the ##w_i## commute with the induced maps ##\mathcal{F}(\phi_{ik})##, and ##A## is defined as precisely the subset of ##\prod A_j## that does this, ##w(W)\subseteq A##. Let ##\rho: \prod A_j\to A## be a left inverse of the inclusion map ##i:A\to\prod A_j##, and let ##w'=\rho\circ w:W\to A##. This map ##w'## is unique because ##w(W)\subseteq A##, so the left inverse would take any element of ##w(W)## to its corresponding element of ##A##.

Is this correct?
 
Yes, seems right!
 
Question: A clock's minute hand has length 4 and its hour hand has length 3. What is the distance between the tips at the moment when it is increasing most rapidly?(Putnam Exam Question) Answer: Making assumption that both the hands moves at constant angular velocities, the answer is ## \sqrt{7} .## But don't you think this assumption is somewhat doubtful and wrong?

Similar threads

  • · Replies 0 ·
Replies
0
Views
2K
  • · Replies 13 ·
Replies
13
Views
3K
  • · Replies 0 ·
Replies
0
Views
835
  • · Replies 4 ·
Replies
4
Views
2K
  • · Replies 2 ·
Replies
2
Views
2K
Replies
6
Views
3K
  • · Replies 1 ·
Replies
1
Views
2K
  • · Replies 3 ·
Replies
3
Views
2K
  • · Replies 5 ·
Replies
5
Views
3K
  • · Replies 2 ·
Replies
2
Views
2K